

Hello,
Dr. Batman
Hello Doctor, Welcome!
Profile

Name: Batman
Email: batman@gotham.com
INFECTIOUS DISEASE
(Total Questions - 159)Q.1. A 62 year old IV drug abuser is brought into the emergency department with complaint of fever, shivering, malaise, shortness of breath and productive cough. Around 8 days ago he developed symptoms consistent with a flu-like illness. Initially there was an improvement in his condition but deteriorated over the past three days. He now has a temperature of 39 C, a pulse of 110 beats/minute, a blood pressure of 100/70 mmHg and a respiratory rate of 22 breaths/minute. A chest x-ray shows bilateral cavitations. What is the most likely causative organism?
Correct Answer : A
Staphylococcus aureus may complicate influenza infection and is seen most frequently in the elderly and intravenous drug users or patients with underlying diseases.
Chest x-ray shows bilateral cavitations. Remember, there is a high incidence of Staphylococcus aureus pneumonia in patients following influenza so in exam if you see a patient with a flu-like illness whose symptoms are now of pneumonia, the likely causative organism is Staphylococcus aureus.
Q.2. A 28 year old man presents with a widespread maculopapular rash over his soles and palms. He also has mouth ulcers. He had a penile ulcer which healed six weeks ago. What is the most likely organism responsible?
Correct Answer : C
Syphilis is a sexually transmitted infection caused by the spirochaete Treponema pallidum. Acquired syphilis is characterized by primary, secondary, and tertiary stages.
The incubation period is around 3 weeks.
Acquired syphilis :
• Chancre - painless ulcer at the site of sexual contact
• Local non-tender lymphadenopathy
• In women, they are found on the vulva, labia, and, much less frequently, on the cervix.
Secondary features :
• Secondary syphilis often appears 6 weeks after the beginning of the primary lesion but may overlap or not appear for several months.
• Systemic symptoms: Fever, lymphadenopathy, headaches, malaise.
• A generalized polymorphic rash often affects the palms, soles, and face
• Papules enlarge into condylomata lata (pink or grey discs) in moist warm areas.
Tertiary features :
• Gummas (granulomatous lesions can occur in any organ but most commonly affect bone and skin)
• Cardiovascular syphilis ascending aortic aneurysms, aortic regurgitation
• Neurological syphilis tabes dorsalis, dementia.
Q.3. A 44 year old HIV positive man complains of a two week history of worsening headache, facial weakness and visual hallucinations. He also reports new onset of eye pain. An MRI head reveals multiple ring shaped contrast enhancing lesions. What is the most likely causative organism?
Correct Answer : A
The symptoms and MRI findings here are highly suggestive of toxoplasmosis, a disease caused by the protozoan Toxoplasma gondii.
The reason behind the HIV history is that toxoplasmosis can reactivate in those with severe HIV disease when their CD4 counts are very low.
Principle manifestations usually include a brain mass lesion, headache, confusion, seizures, and focal neurologic deficits. These symptoms occur due to increased intracranial pressure.
In patients with AIDS, cerebral toxoplasmosis should be right at the top of the list of differentials if one presents with focal neurological symptoms.
Imaging such as a CT or MRI scan of the head would show a "ring" (contrast) enhancing lesion with oedema. Scans may show solitary lesions.
Given the choice of imaging modality, choose an MRI over a CT as MRI appears to be more sensitive than CT in imaging for cerebral toxoplasmosis.
TOXOPLASMOSIS : Toxoplasma gondii is a protozoa that infects the body via the gastrointestinal tract, lung, or broken skin. Human infection occurs from consumption of undercooked meat or, foot or water contaminated with the oocytes.
Its oocysts release trophozoites which migrate widely around the body including to the eye, brain, and muscle.
The usual animal reservoir is cats where oocytes are produced in the cat’s intestines and shed in its faeces.
This is thought to be an evolutionary adaptation to help toxoplasma gondii complete its life cycle: rats are exposed to cat faeces and are infected with toxoplasma gondii. Most infections are asymptomatic.
Symptomatic patients usually have a self-limiting infection, often having clinical features resembling infectious mononucleosis (fever, malaise, lymphadenopathy). Other less common manifestations include meningoencephalitis and myocarditis.
Treatment: Pyrimethamine plus sulphadiazine.
Q.4. A 25 year old man was admitted to the hospital with a fever of 38.8 C and rigors. He complains of a three day history of general malaise, mild headaches, arthralgia and myalgia. He especially complained of bilateral calf pain. His past history is significant for a trip to the Caribbean from which he returned five days ago. He describes the trip to the Caribbean with his friends as being amazing and describes a host of activities such as swimming, hiking and fishing. Upon further questioning, he also revealed that he noticed a red discoloration of his eyes before they turned to yellow. The patient also mentions seeing “spots” on his skin. On physical examination, the patient appears to be jaundiced. Abdominal examination revealed generalized abdominal tenderness. His vitals are as follows: Blood pressure 98/56 mmHg Heart rate 122 beats per minute Respiratory rate 18 breaths per minute Oxygen saturation on room air 98% Blood tests were done and the laboratory results are as follows: Alanine transferase (ALT) 89 U/L (5-35 U/L) Aspartate transaminase (AST) 60 U/L (5-35 U/L) Alkaline phosphatase (ALP) 162 U/L (30-150 U/L) Gamma glutamyl transferase (GGT) 33 U/L (8-60 U/L) White blood cells 15 x 109/L (4-11 x 109/L) Platelets 180 x 109/L (150-400 x 109/L) What is the most appropriate investigation in this scenario to confirm the diagnosis?
Correct Answer : B
This patient is suffering from leptospirosis.
Leptospirosis is the most widespread zoonotic infection worldwide. The main risk factor for acquiring the infection is direct or indirect contact with the urine of infected animals. Contact can occur secondary to occupational exposure or accidentally via exposure to unclean sources of water.
Travelers and athletes participating in water sports, such as those who compete in triathlons, represent a growing population at risk. Ask for a history of rowing, fishing, contact with animals, or travel.
The red discoloration of the eye seen initially is subconjunctival hemorrhage which occurs in more than 90% of patients with leptospirosis.
Aminotransferase levels are usually elevated but they are rarely higher than 200 U/L just as seen in this stem. The single most appropriate investigation in this scenario would be blood and urine cultures. This is because leptospirosis can be isolated from blood during the initial phase for the first seven to ten days of the disease.
The Paul Bunnell test or the mono spot test is a form of the heterophile antibody test. It is a rapid test for infectious mononucleosis due to the Epstein-Barr virus (EBV). This choice is therefore incorrect.
Q.5. A 82 year old man was brought into the emergency department with a low level of consciousness. His wife mentions that he had a severe headache for the last 20 hours and was very sensitive to light. He has a temperature of 39.0 C, a pulse of 118 beats/minute, a blood pressure of 80/55 mmHg and a respiratory rate of 32 breaths/minute. He is conscious but confused. Kernig’s sign was positive. High flow oxygen and IV fluids was immediately started. What is the most appropriate immediate action?
Correct Answer : D
This patient is having meningitis. Treatment for meningitis should be started before doing any investigations due to the seriousness of the disease. In a hospital setting, give intravenous third-generation cephalosporin antibiotics (ceftriaxone or cefotaxime).
Q.6. A 23 year old man has been using intravenous drugs. You explain to him the dangers of this, including the risk of contracting a blood borne virus such as Hepatitis B. Which of the following laboratory tests will first become abnormal after acquiring hepatitis B infection?
Correct Answer : A
Hepatitis B surface antigen (HBsAg) is a marker of acute hepatitis B infection and the first abnormal serological test.
Hepatitis B e antigen (HBeAg) and Hepatitis B DNA also appear soon after. The presence of HBeAg in the serum of patients can serve as a marker of active replication.
Hepatitis B core IgM antibody indicates recent infection with hepatitis B virus (< 6 months). It appears shortly after the onset of symptoms and at a similar time to the rise in alanine aminotransferase (ALT).
The presence of anti-hepatitis B e antibody (HbeAb) is a sign that the patient has moved into a non-replicative stage of chronic hepatitis B – usually due to treatment. It is not used as a marker of acute hepatitis B.
Hepatitis B is a DNA virus spread through contact with infected bodily fluids.
Key summary of the most commonly asked hepatitis B serology that is a must-know for the exam:
• HBsAg positive – Infection acute or chronic
• HbsAg positive and HbeAg positive – Highly infectiousness
• HBsAg positive alone – Recent vaccination
• Anti-HBc positive – Past infection
Q.7. A 49 year old man with known HIV presents with history of cough and shortness of breath. His CD4 count is measured at 350 mm3. A chest x-ray was performed and shows lobar consolidation. He has a temperature of 38.1 C, a respiratory rate of 30 breaths/minute and a heart rate of 90 beats/minute. What is the most likely causative organism?
Correct Answer : C
At first, the impression that is given in the stem points towards Pneumocystis jiroveci given the history of cough and an HIV-positive patient. Pneumocystis jiroveci is the most common opportunistic infection in HIV-positive patients.
However, one must remember that Pneumocystis jiroveci tends to affect HIV-positive patients who have a CD4 count below 200 cells/mm³. For this reason, all patients with a CD4 count < 200/mm³ should receive Pneumocystis jiroveci prophylaxis.
While it is true that Pneumocystis jiroveci can also infect an HIV-positive patient with a CD4 count above 200 cells/mm³, it is less likely compared to Streptococcus pneumoniae.
The chest X-ray findings also point more toward Streptococcus pneumoniae as the causative organism. Remember that Streptococcus pneumoniae is the most common causative organism of lobar pneumonia and it accounts for around 80% of cases of community-acquired pneumonia (CAP).
Note that, HIV infection is also a risk factor for Streptococcus pneumoniae infections.
Q.8. A 41 year old lady is suffering from fever, sore throat and loss of appetite. Her symptoms have been persistent and worsening over the past few weeks. She has had a recent renal transplant and is currently on immunosuppressive medication. Serology was taken for toxoplasmosis and has returned with the presence of IgE antibodies which is interpreted as having a recent acquired infection. Her renal function is satisfactory. What is the most appropriate treatment?
Correct Answer : D
Immunocompetent patients require treatment if symptoms of toxoplasmosis are severe and persistent. Pyrimethamine is the most effective agent for toxoplasmosis infections. This is given in combination with sulfadiazine. Sometimes clindamycin is used instead of sulfadiazine.
Q.9. A 33 year old man presents with an erythematous patch on his thigh, which has been enlarging in the last few days. He went for a camping trip a week ago. He has no allergies to any known medication. He is otherwise asymptomatic. What is the most appropriate management?
Correct Answer : C
Lyme disease is caused by the spirochaete Borrelia burgdorferi and is spread by ticks. Tick needs at least 24 hours of attachment to transmit the Borrelia burgdorferi organism.
Symptoms begin 3-30 days after the bite of the tick. Patients develop the erythema migrans rash at the site of the bite. (An erythematous patch, which may enlarge in the first few days).
Even without treatment, the rash resolves in several weeks. A flu-like illness with fever, chills, and myalgias occurs in half of patients. Neurologic symptoms develop several weeks later. This is most commonly paralysis of the seventh cranial nerve (facial paralysis) and may be bilateral.
Meningitis and encephalitis may develop as well. Cardiac symptoms develop and are most commonly an AV heart block or Myocarditis. Joint involvement may develop but this is months to years later.
Investigation :
Serology: Antibodies to Borrelia burgdorferi
Management :
• Doxycycline if early disease. Amoxicillin is an alternative if doxycycline is contraindicated (e.g. pregnancy)
• Ceftriaxone if a disseminated disease.
Q.10. A 55 year old man has auricular pain and tinnitus on his left ear. On inspection, a painful vesicular rash around the auditory canal is noted. He also has decreased hearing on the left ear. What is the most likely diagnosis?
Correct Answer : A
Ramsay Hunt syndrome (herpes zoster oticus) is caused by the reactivation of the varicella-zoster virus in the geniculate ganglion of the seventh cranial nerve.
Features :
• Auricular pain is often the first feature
• Facial nerve palsy (ipsilateral facial palsy, loss of taste)
• Painful vesicular rash around the ear on the auditory canal
• Vertigo and tinnitus
• Ipsilateral hearing loss
Management :
• Oral acyclovir and corticosteroids are usually given
• For herpetic neuralgia, give amitriptyline for the pain.
Q.11. A 33 year old male patient presents with white patches in the mouth that can be wiped off and is easily removed leaving behind a red base which is painless. He has cracks at the corners of his mouth. What is the most likely diagnosis?
Correct Answer : C
This is typical for oral candidiasis.
White patches in the mouth can be easily removed, leaving an underlying red base that is usually painless.
PSEUDOMEMBRANOUS ORAL CANDIDIASIS (ORAL THRUSH) :
• Curd-like white patches in the mouth
• White pseudomembrane can be easily removed, leaving an underlying red base that is usually painless (in contrast with leukoplakia, which cannot be rubbed off)
• Cracks can occasionally be seen at the corners of the mouth
Predisposing factors :
• Inhaled corticosteroids – risk can be decreased by using large volume spacer or mouthwash after use.
• Diabetes mellitus
• Recent use of broad-spectrum antibiotics
• Elderly with false teeth
Treatment :
• Oral fluconazole
• NICE CKS recommends using miconazole oral gel as first-line or nystatin suspension if the infection is mild and localized.
Q.12. A 73 year old woman living in a nursing home, presents with rashes on her finger webs and also at her axillary folds. She complains of itching which is more severe at night. What is the most appropriate management?
Correct Answer : B
In exam, when you see an elderly living in a nursing home with rashes, think of scabies Permethrin 5% is probably the only treatment that would be asked in exam for scabies.
Scabies is a parasitic skin infection characterized by superficial burrows, intense pruritus, and secondary infections Aetiology Sarcoptes scabiei. Transmitted by skin-to-skin contact.
Clinical Findings : Pruritus, burrows, papules, commonly found on flexor surfaces of wrists, finger webs, elbows, axillary folds, areola of the breast in women, and genitals of the males. Scabies dig into the skin at the skin folds.
• Patients who are immunocompromised or debilitated may develop a severe form of scabies called Norwegian scabies (crusted scabies). These patients present with diffuse cutaneous involvement with crusting and malodorous discharge.
Treatment :
• Scabies treatment is with permethrin 5% which is first-line
• Malathion 0.5% is second-line
• Note: All household and close physical contacts should be treated at the same time, even if asymptomatic.
Q.13. A 75 year old woman has weakness of the left side of her face and pain deep within the left ear. The ear pain was paroxysmal at first but after a day it became constant and radiates into the pinna. Now she says she hears ringing in her left ear. A mild left hearing loss was noted. There are blisters on the skin of the ear canal and auricle. What is the most likely diagnosis?
Correct Answer : D
Please see Q - 10
Q.14. A 12 month old child who is HIV positive is due for his measles, mumps, and rubella (MMR) vaccine. His CD4 count is more than 800 cells/mL. What is the most appropriate action?
Correct Answer : C
Even though the measles, mumps, and rubella (MMR) vaccine contains live attenuated viruses, it is sometimes recommended for people with HIV/AIDS. It is contraindicated if the patient is severely immunocompromised including an HIV-infected patient with CD4 counts less than 200 cells/mL (or less than 750 cells/mL in children).
Q.15. A 38 year old woman recently returned from Bangkok. She did not have any malaria prophylaxis before leaving the UK on her trip. She presents with a high fever, generalized macular blanching rash, tender and swollen cervical lymphadenopathy and generalized myalgia. What is the most likely diagnosis?
Correct Answer : B
Here is a summarized list of the presentation of these diseases.
Remember to tie in the signs and symptoms with the travel history :
• Malaria: fever, chills, rigors
• Dengue Fever: generalized rash, biphasic fever, retro-orbital pain
• Typhoid: severe headache, patients adopt a crouching position
• Diphtheria: presents with flu-like symptoms initially, enlarged anterior cervical lymph nodes.
Q.16. A 45 year old man has developed an annular rash with a scaly edge on his thigh. The rash has been spreading over the last 3 weeks. He also complains of general aches and pains. What is the most appropriate investigation?
Correct Answer : C
Annular rash is seen in Lyme disease.
Q.17. An 8 week infant has been diagnosed as HIV positive. What is the most appropriate immunization plan for the infant?
Correct Answer : A
BCG should not be given to HIV-positive patients. All other vaccinations can be given. MMR vaccinations should not be given if the CD4 count is below 200 cells/mL.
As the question stem does not state the CD4 count, administering all vaccines as scheduled except the BCG vaccine is most appropriate.
There are debates about avoiding live immunization for HIV in children however it is best to follow the guidelines.
The two vaccines that HIV-positive patients should NOT have (regardless of CD4 count) are: BCG & Yellow fever.
Q.18. A 32 year old man has recently been to Thailand and returned with a sore throat, cervical lymphadenopathy, malaise and a mild fever. On examination of the throat, there is whitish exudate covering the tonsils. What is the most likely infectious agent causing his symptoms?
Correct Answer : B
Sore throat and cervical lymphadenopathy are some of the main features of infectious mononucleosis. If this was an HIV infection, there would be more of a generalized lymphadenopathy.
Infectious mononucleosis (glandular fever) is caused by the Epstein-Barr virus (also known as human herpesvirus 4, HHV-4). It is most common in adolescents and young adults.
Features :
• Sore throat; tonsillar enlargement is common, classically exudative and maybe massive
• Palatal petechiae
• Lymphadenopathy, especially neck glands
• Pyrexia
• Malaise
• Splenomegaly - may rarely predispose to splenic rupture
• A maculopapular, pruritic rash develops in around 99% of patients who take ampicillin/amoxicillin whilst they have infectious mononucleosis. Thus, they should not be given to any patient who might have infectious mononucleosis.
Diagnosis :
• Heterophil antibody test (Monospot test / Paul Bunnell)
Other investigations :
• FBC
• Raised white cell count with lymphocytosis and a relative atypical lymphocyte count greater than 20%
• ESR is elevated
- Management is supportive. Simple analgesia for any aches or pains.
Q.19. A 55 year old immunocompromised patient presents with dysphagia and pain on swallowing. He has a redness, fissuring and soreness at the angle of his mouth. What is the causative organism?
Correct Answer : C
Angular cheilitis
• Redness, fissuring, and soreness at the angle of the mouth.
• It can be due either to Candida spp. or to bacterial infection (mainly S. aureus)
• Contributing factors are older age, ill-fitting dentures, immunocompromised, vitamin B12 deficiency, or iron-deficiency anaemia.
Q.20. A 4 year old child is brought by his parents with a clean wound on his leg. He has never been immunized as his parents were worried about the side effects of the immunizations in the past. There are no contraindications to immunization and the parents of the child are now open to the idea. The child’s parents would like advice on the best method to reduce risk of infections. What is the most appropriate action?
Correct Answer : D
A full course of DPT vaccine would be appropriate to prevent tetanus in the future. Children aged under 10 years if miss the vaccine can resume the schedule with the primary immunization (first 3 doses). There should be a minimum of one month between doses of the vaccine.
This is then followed by the first booster immunization which is 3 years after primary immunization in children under 10 years of age. The second booster immunization is given 10 years after the first booster.
TETANUS PROPHYLAXIS :
Tetanus vaccine is currently given as part of the routine immunization schedule at:
• 2 months
• 3 months
• 4 months
• 3-5 years
• 13-18 years
If high-risk wound - Give intramuscular tetanus immunoglobulin irrespective of whether 5 doses of tetanus vaccine have previously been given.
If incomplete or unknown vaccination - Give a complete course of tetanus vaccine.
What is considered a high-risk wound?
• Wounds contaminated with soil
• Compound fractures
• Wounds containing foreign bodies
• Wounds or burns in people with systemic sepsis. After the 5th dose of the tetanus vaccine, immunity is considered for life and you do not need any further boosters.
Q.21. A 26 year old woman presents to her GP with a headache, photophobia and a fever. On examination, a generalized rash that does not blanch on pressure was observed. What is the most appropriate initial management?
Correct Answer : C
She has signs of meningitis. If you suspect meningitis and the patient is not yet in the hospital give IM benzylpenicillin and send the patient to the hospital urgently. In a hospital setting, give intravenous third-generation cephalosporin antibiotics.
If this same question was given, but there was a diagnosis of Listeria, then IV amoxicillin and gentamicin would be the correct answer.
If this same question was given, but she is penicillin or cephalosporin allergic, then chloramphenicol would be the answer.
Investigations :
Generally, treatment for meningitis should be started before doing any investigations due to the seriousness of the disease. Rash If the patient has a rash, then perform a blood culture as the diagnosis is most likely meningococcal septicemia. The causative organism is Neisseria meningitides.
If there is no rash then a lumbar puncture would be a better answer, but this can only be done if there are no signs of raised intracranial pressure (ICP).
Patients with increased ICP from mass lesions often display decreased levels of consciousness, focal neurological signs, or papilloedema on physical exams.
Q.22. A 15 year old boy had a patchy rash over his body following antibiotic treatment for sore throat. On examination, he has cervical lymph node enlargement and mild splenomegaly which is tender on palpation. What is the most likely antibiotic that would have caused the rash?
Correct Answer : A
A maculopapular, pruritic rash develops in around 99% of patients who take ampicillin / amoxicillin whilst they have infectious mononucleosis.
Q.23. A 33 year old man has complaints of dysuria and three tender penile ulcers. He is sexually active and does not use any protection. What is the most likely diagnosis?
Correct Answer : B
Genital herpes simplex which is caused by herpes simplex virus may present with painful multiple ulcers and also dysuria.
Syphilis ulcers are usually painless and thus is not the correct answer.
GENITAL HERPES :
May be asymptomatic or may remain dormant for months or even years. When symptoms occur soon after a person is infected, they tend to be severe. They may start as multiple small blisters that eventually break open and produce raw, painful sores that scab and heal over within a few weeks.
The blisters and sores may be accompanied by flu-like symptoms with fever and swollen lymph nodes. Often also accompanied by dysuria. Genital herpes can be a chronic, lifelong infection.
HSV-1 was known for its orolabial herpes (cold sores) and HSV-2 for its genital herpes. But now, HSV-1 is the most common cause of both orolabial herpes and genital herpes.
Signs: Flu-like prodrome, then grouped vesicles/papules develop around genitals. These burst, and form shallow ulcers.
Management: Oral acyclovir. Some patients with frequent exacerbations may benefit from longer-term acyclovir.
Q.24. A 34 year old man who has a new diagnosis of haematological malignancy presents in the emergency department with bruises all over his abdomen. He has a temperature of 38.6 C. His respiratory rate is 25 breaths/minute, heart rate is 102 beats/minute and blood pressure is 80/50 mmHg. His blood results show: White cell count 23 x 109/L Neutrophils 0.4 x 109/L He is commenced on meropenem. What is the most likely diagnosis?
Correct Answer : C
A temperature of more than 38 C and neutrophil count of less than 0.5 x 109/L defines neutropenic sepsis.
NEUTROPENIC SEPSIS :
Neutropenic sepsis is a potentially fatal complication of chemotherapy.
Febrile neutropenia is defined as:
• An oral temperature 38.5°C or two consecutive readings of 38.0°C for two hours and
• An absolute neutrophil count of 0.5 x 109/L
Chemo is the most common cause of neutropenic sepsis
General Management for Neutropenic Sepsis :
• Antibiotics must be started immediately (do not wait for the WBC)
• Start intibiotic therapy with piperacillin with tazobactam immediately
Q.25. A 24 year old man presents with a deep penetrating wound on his foot after having stepped on a nail in a field. The wound is deep. He does not remember if he had tetanus vaccines when he was a child. What is the most appropriate management to be given?
Correct Answer : D
If uncertain history of previous vaccination and high-risk wound (like in this case), give vaccine and tetanus immunoglobulin (TIG). This should be a complete course.
Q.26. A 16 year old girl has a sore throat. She feels tired and weak. Oropharyngeal examination shows tonsillar enlargement which is exudative. Her GP prescribed her amoxicillin after which she developed a pruritic rash. What is the most likely diagnosis?
Correct Answer : C
A maculopapular, pruritic rash develops in around 99% of patients who take ampicillin/amoxicillin whilst they have infectious mononucleosis.
Q.27. A 16 year old girl attends clinic with a sore throat and palatal petechiae. A full blood count was done which shows: Haemoglobin 109 g/L White cell count 25 x 109/L Platelets 88 x 109/L A Paul Bunnell test was shown to be positive. What is the most likely diagnosis?
Correct Answer : B
The clincher here is the positive Paul Bunnell test which is only seen in infectious mononucleosis (Glandular fever).
The rest of the symptoms fit perfectly as well. Palatal petechiae and sore throat can occur with infectious mononucleosis. WBCs are classically high and the anaemia is secondary to cold agglutinins (IgM).
Q.28. A 35 year old lady presents with recurrent extremely painful ulcers on her vulva. Viral culture and DNA detection using polymerase chain reaction (PCR) of a swab from the ulcer has come back as negative. What is the most appropriate investigation which will lead to the diagnosis?
Correct Answer : C
The probable diagnosis here is Genital Herpes Simplex.
Usually, Viral culture and DNA detection using polymerase chain reaction (PCR) of a swab from the base of an ulcer are used to diagnose genital herpes. Now NAAT testing is the preferred diagnostic modality above PCR-based methods but both NAAT testing and PCR-based methods are more sensitive than viral culture.
Anti-HSV antibodies are only used in certain scenarios. One of them is if there are recurrent/atypical genital ulcers with negative culture or PCR results. Remember, serum HSV-specific antibodies should not be routine as this blood test cannot reliably differentiate between a recent or a previous infection. It may be helpful in specific situations like in this case.
Q.29. A 54 year old smoker presents to Accident & Emergency with the complaint of a productive cough. He says that he coughs up about a tablespoon full of yellow sputum every day. He also complains of chest tightness and mentions noticing a fever as well. These symptoms have been present for the past 5 days. He smokes two packs of cigarettes per day and has been smoking for the past 20 years. Physical examination reveals the presence of herpes labialis on the patient’s lower lip. The patient’s observations are as follows: Temperature 38.7 C Blood pressure 150/95 mmHg Heart rate 95 beats per minute Respiratory rate 20 breaths per minute Oxygen saturation 96% on room air. Auscultation of his chest reveals breath sounds that are distant but clear with crackles heard at the right lung base. A chest X-ray that was subsequently done reveals a right lower zone consolidation. What is the most likely organism responsible for causing pneumonia in this patient?
Correct Answer : A
This patient has the typical symptoms of pneumonia as well as the presence of herpes labialis. That, in addition to the consolidation seen on the chest X-ray, is very suggestive of a streptococcal infection which is a gram-positive diplococci (found in pairs).
Streptococcus pneumoniae is the commonest bacterial pneumonia and one of the clinical features that separates it from the rest of pneumonias is herpes labialis.
Q.30. A 36 year old jewish man presents with multiple purple papular lesions on his face and upper trunk measuring 1-2 cm across. It has been slowly growing over the last couple of years. It is not painful or itchy. What is the most likely diagnosis?
Correct Answer : D
Kaposi's sarcoma (KS) is a connective tissue cancer caused by human herpesvirus 8 - now called Kaposi's sarcoma-associated herpesvirus (KSHV). The malignant lesion is characterized by neoplastic cells and abnormally growing blood vessels.
KS is different from other neoplasms because lesions may begin in more than one place at the same time.
Types of Kaposi's sarcoma:
1. Classic: especially elderly Jewish or Mediterranean man. It is rare and progresses slowly over the years.
2. Endemic or African KS: affects young adult men who live near the African equator and have a normal immune system.
3. KS in immunosuppression: Aggressive course with visceral involvement.
4. AIDS-related: May be life-threatening with many skin, gut, and lung lesions.
- It affects mostly homo- or bisexual men
Presentation :
• Skin lesions may be nodular, papular, or blotchy; they may be red, purple, brown, or black.
• Lesions can also be seen under or on mucous membranes, with similar characteristics.
• The most common sites include the mouth, nose, and throat.
• Usually painless but may become painful if inflamed or swollen.
• Lesions may also involve internal organs - eg, lungs (leading to dyspnoea), gastrointestinal tract (it can cause fatal bleeding), and lymphatics, resulting in lymphoedema.
Q.31. A 35 week pregnant woman presents to the Antenatal Day Unit with productive cough and rigors. This is her first pregnancy and there have been no issues to date. She returned from Uganda two weeks ago from a family visit. She is suspected to have respiratory tuberculosis. What is the most likely medication that should not be used in pregnancy?
Correct Answer : B
The standard unsupervised six-month treatment regimen may be used during pregnancy. Streptomycin should not be used in pregnancy because it has been shown to have harmful effects on the fetus. This patient should be treated as the risks of untreated tuberculosis are greater in pregnancy than in the medication.
The risk of the pregnancy are perinatal infection, low birth weight or growth retardation, and premature delivery.
Treatment for tuberculosis is the same for pregnant and non-pregnant status: 4 drug therapy (Isoniazid, Rifampicin, Pyrazinamide, Ethambutol).
Q.32. A 36 year old lady with Hodgkin’s lymphoma has chemotherapy 8 days ago. She presents with a temperature of 39.5 C and left sided abdominal pain. Her pulse rate is 96 beats/minute. Full blood count was sent and bloods were taken for culture. What is the most appropriate next action?
Correct Answer : A
There are clear signs of infection with the risk that the patient could be having neutropenic sepsis thus broad spectrum IV antibiotics should be started empirically while waiting for blood reports.
There are 2 main reasons neutropenia is seen in lymphoma:
1. Lymphoma in the bone marrow
• If lymphoma cells are in the bone marrow, they take up space that is normally used to produce healthy blood cells which can lead to neutropenia.
2. Side effects of treatment
• Although treatment aims to kill the lymphoma cells, a side effect of many types of chemotherapy, and some radiotherapy treatments (eg radiotherapy to the pelvis), is that some healthy cells are also destroyed.
This may include blood cells that are developing in the bone marrow.
• Depending on the strength of your chemotherapy regimen, neutropenia is most commonly seen 10 to 14 days after chemotherapy.
Q.33. A 25 year old woman presents with a painful shallow ulcer on the vulva. It is associated with painful inguinal lymphadenopathy. She is sexually active. What investigation is likely to identify the cause of the ulcer?
Correct Answer : A
A single painful ulcer is likely to be Chancroid, caused by Haemophilus ducreyi. The pain inguinal lymphadenopathy is also typical.
As the saying goes, “You do cry with ducreyi”.
Primary syphilis typically causes a painless ulcer (chancre). Herpes simplex virus (HSV) leads to several inflamed papules and vesicles.
It would be uncommon to have a single ulcer. In addition, the presence of HSV antibodies does not confirm the diagnosis (only past exposure), as many patients will acquire the infection but remain asymptomatic. Blood and urine cultures are not required.
Q.34. A 7 year old school boy has been diagnosed with meningococcal meningitis. What is the most appropriate prophylactic management?
Correct Answer : B
Meningococcal Prophylaxis -
Prevention of secondary cases of meningococcal meningitis is with rifampicin or ciprofloxacin. It is usually given to all intimate, household, or daycare contacts who have been exposed to the patient within 10 days of onset.
Q.35. A 74 year old female presents with headache and neck stiffness to the emergency department. Following a lumbar puncture, the patient was started on IV ceftriaxone. CSF culture reports as having Listeria monocytogenes. What is the most appropriate treatment?
Correct Answer : C
The therapy after identification of Listeria monocytogenes in CSF is ampicillin 2g 4 hourly + gentamicin 5 mg/kg divided into 8-hourly doses.
Q.36. A 13 year old girl complains of a 3 day history of hoarseness of voice associated with dry cough. She presents with a fever and malaise. On direct laryngoscopy, her vocal cords are seen to be grossly oedematous. What is the most appropriate investigation?
Correct Answer : D
This is likely a case of a common cold. No further investigations or management is required.
Q.37. A 20 week pregnant lady presents with intermittent fever, coughs, headaches, myalgia, gastric upset and mild confusion. She arrived from Ghana 12 days ago. She travelled there to visit her family. Before she left, she was prescribed chloroquine and proguanil and she took it as prescribed starting 1 week before entering Ghana. She is still taking chloroquine and proguanil as her doctor had asked her to continue it for 4 weeks. On examination, she has a yellowish tinge on her skin. What is the most likely diagnosis?
Correct Answer : A
This lady has been taking malaria chemoprophylaxis but it is important to note that Malaria can still occur in people even though they take Malaria chemoprophylaxis. Chloroquine and proguanil can be used in pregnancy but these drugs are not appropriate for most areas because their effectiveness has declined, particularly in Sub-Saharan Africa.
Key message: Chloroquine and proguanil are safe in pregnancy, but they are no longer efficacious in areas of chloroquine resistance.
Mefloquine is now the drug of choice and recommended by the Royal College of Obstetricians and Gynaecologists (RCOG) for prophylaxis in pregnant women in the second and third trimester traveling to chloroquine-resistant areas.
Evidence also suggests that pregnant women are twice as likely as non-pregnant women to be bitten by anopheline mosquitoes which is the reason we should encourage women not to travel to areas with malaria.
Clinical features that are seen with Malaria include:
• Intermittent fevers
• Chills
• Rigors
• Headache
• Cough
• Myalgia
• Gastrointestinal upset
• Splenomegaly
• Hepatomegaly
• Jaundice
Q.38. A 5 year old boy was brought to his GP with a temperature of 38.8 C and numerous pruritic vesicles on his chest and back. They appear in circular crops and initially started on his face before spreading to his chest and back. What is the most appropriate management?
Correct Answer : C
The diagnosis here is chicken pox. The management here would involve reassurance, and supportive management like paracetamol for the fever, antihistamine, and calamine lotion for the pruritus. There is no role of acyclovir in a healthy child or adult.
Antibiotics are administered only in the case of superimposed infection.
Chickenpox :
• Caused by primary infection with varicella-zoster virus. Reactivation of the dormant virus after a bout of chickenpox leads to herpes zoster (shingles)
• Highly infectious and its spread is via the respiratory route. Most chickenpox is mild to moderate and self-limiting but serious complications can occur in immunocompromised patients.
Clinical features :
• Pyrexia : Often the first feature
• Itchy, rash starting on the head, chest, and back before spreading
• Lesions are usually most concentrated on the chest and back
• Initially rash is macular then papular then vesicular then dry crust
Clinical features tend to be more severe in adults
Management :
• Pruritus managed by sedating antihistamines and emollients. While some sources mention that calamine lotion is no longer recommended, as when it dries it ceases to be effective, it is still used and the advice that is given is to reapply it regularly.
• Administer varicella zoster immunoglobulin (VZIG) to: Immunocompromised with exposure or Newborn with peripartum exposure or Pregnant women with exposure and with no varicella antibodies.
• Administer acyclovir to: Pregnant women who develop chickenpox. Immunocompromised who develop chickenpox.
Q.39. A 78 year old nursing home resident has intensely itchy rash. White linear lesions are seen on the wrists and elbows, and red papules are present on his penis. What is the most appropriate management?
Correct Answer : C
When you see an elderly living in a nursing home with rashes, think of scabies. Permethrin 5% is probably the only treatment that would be asked in exam for scabies.
Q.40. A 5 year old boy has a sudden onset of fever and bilateral swelling at the angles of the jaw. He has ear pain when he chews. The GP saw him yesterday for bilateral parotid pain and prescribed him paracetamol. He currently has a temperature of 38.8 C. What is the most appropriate next step?
Correct Answer : D
Remember that there is no specific treatment for mumps but drugs such as paracetamol and ibuprofen may give symptomatic relief. In this case, reassurance is all that is needed. Mumps is a self-limiting condition.
Mumps :
• Mumps is an acute, generalized infection caused by a paramyxovirus, usually in children and young adults
• It can infect any organ but usually affects the salivary glands
• The virus is highly infectious with transmission by droplets spread in saliva via close personal contact
• Infected persons excrete the virus for several days before symptoms appear and for several days afterward
Presentation :
• Mumps can be asymptomatic
• Nonspecific symptoms lasting a few days, such as fever, headache, malaise, myalgia, and anorexia, can precede parotitis
• Parotitis is usually bilateral although it can be unilateral
• Typically, there is pain at or near the angle of the jaw
• Fever may be as high as 39.5°C without rigors in small children
• Swelling causes distortion of the face and neck with skin over the gland hot and flushed but there is no rash
• With severe swelling, the mouth cannot be opened and is dry because the salivary ducts are blocked.
• Discomfort lasts for three or four days but may be prolonged when one side clears and the other side swells
• Usually just the parotid glands are involved but, rarely, the submaxillary and sublingual salivary glands are affected Orchitis. Orchitis may occur four or five days after the start of parotitis but it often appears without it. This can sometimes lead to the diagnosis being missed.
Orchitis presents with chills, sweats, headache, and backache with swinging temperature and severe local testicular pain and tenderness. The scrotum is swollen and oedematous so that the testes are impalpable. Orchitis is usually unilateral but may be bilateral.
Q.41. A 33 year old known drug abuser has swelling and erythema in his arm where he injects. He has a fever and appears sick. He is asking for morphine as the pain is severe and seems to be disproportionate to the clinical appearance. Bullae is seen on the skin of his arm. He was started on intravenous flucloxacillin but the infection has not responded to antibiotics and seems to be worsening. What is the most likely diagnosis?
Correct Answer : B
Necrotizing fasciitis is a life-threatening infection that rapidly spreads caused predominantly by group A-hemolytic Streptococci. It is defined as a necrotizing infection involving any layer of the deep soft tissue compartment (dermis, subcutaneous tissue, fascia, or muscle).
Risk factors :
• Intramuscular or subcutaneous drug injection
• Diabetes
• Immunosuppression
Presentation according to days :
• Day 1 to 2 - Swelling, erythema, pain over the affected area (mimics cellulitis)
- Margins of infection are poorly defined, with tenderness extending beyond the apparent area of involvement (unlike cellulitis)
- No response to antibiotics (unlike cellulitis)
• Day 2 to 4 - Bullae, indicating skin ischaemia (unlike cellulitis). Skin progresses to grey colour due to necrosis (unlike cellulitis). Subcutaneous tissues have a wooden-hard feel (unlike cellulitis). From intense pain to anaesthesia-like pain due to nerves being destroyed.
• Day 4 to 5 - Septic shock develops. As necrotizing infection is deep within the skin and is not visible, it is often difficult to diagnose. One important notable feature is if the pain is severe, and disproportionate to the physical signs, think necrotizing fasciitis.
Q.42. A 33 year old lady was at high-risk for Hepatitis-B and was started on vaccination. After 2 injections, she was found to be pregnant. What is the most likely serological result during antenatal screening to suggest vaccine immunization?
Correct Answer : D
HbsAg-negative; anti HBc-negative; anti-HBs-negative : Susceptible
HbsAg-negative; anti-HBc-positive; anti-HBs-positive : Immune due to natural infection
HbsAg-negative; anti-HBc-negative; anti-HBs-positive : Immune due to Hep-B vaccination
HbsAg-positive; anti HBc-positive; IgM anti-HBc-positive; anti-HBs-negative : Acutely infected
HbsAg-positive; anti HBc-positive; IgM anti-HBc-negative; anti-HBs-negative : Chronically infected
Q.43. A 5 year old child started having a fever two days ago associated with neck stiffness, chills, impaired consciousness and vomiting. He has a history of travel with his parents to Ghana and returned 6 weeks ago. Before he left to Ghana, he was started on malaria prophylaxis. A full blood count shows that the young child is anaemic. What is the most likely diagnosis?
Correct Answer : C
Malaria prophylaxis does not provide full protection against all subtypes of malarial parasites. The two top choices given here are cerebral malaria and meningococcal meningitis.
The full blood count showing anaemia points towards cerebral malaria as the diagnosis rather than meningococcal meningitis. Impaired consciousness is one of the signs of cerebral malaria.
There are no specific symptoms of malaria so it is critical to consider the possibility of the diagnosis.
Q.44. An 8 year old boy has red, itchy rash on his abdomen, face, arms and legs that has turned into fluid-filled blisters. A few days later they crusted over. What is the main mode of transmission of this condition?
Correct Answer : A
The diagnosis here is chicken pox. It is caused by the varicella-zoster virus. The mode of transmission of the varicella-zoster virus is mainly person-to-person by airborne respiratory droplets but also occurs by direct contact with the vesicle fluid of chickenpox cases or contact with the vesicle fluid of patients with herpes zoster.
Q.45. A 34 year old man from Egypt is admitted with abdominal pain to the emergency department. An abdominal X-ray reveals bladder calcification and evidence of obstructive uropathy. What is the most likely causative organism?
Correct Answer : B
Bladder involvement is caused by Schistosoma haematobium while Schistosoma mansoni is mainly responsible for intestinal forms of disease. Urinary schistosomiasis (bilharzia). This is caused by the parasitic trematode (or flatworm) called Schistosoma haematobium. It occurs in Africa, Egypt and the Middle East.
Q.46. A 33 year old lady who works at a nursing home presents with itching. On examination, linear tracks on the wrist are seen. She says that 2 days ago she had come in contact with a nursing home resident with similar symptoms. What is the most likely mechanism of itching?
Correct Answer : A
In scabies, pruritus develops as an allergic reaction to infection, around 4-6 weeks after infestation. Most of the symptoms of scabies are due to your immune system's response to the mites, their saliva, their eggs, or their faeces. In other words, the rash and the itching are mostly caused by your body's allergic-like reaction to the mites, rather than the mites themselves.
Q.47. A 24 year old male has a history of urethral discharge and dysuria. He is sexually active with other men and has had four sexual partners in the last year. He does not practice safe sex. Urethral swabs were taken which results came back positive for chlamydia. What is the most likely complication if left untreated?
Correct Answer : C
Chlamydia is the sexually transmitted infection and is caused by Chlamydia Trachomatis.
Males tend to have either:
• Classical urethritis with dysuria and urethral discharge
• Epididymo-orchitis presenting as unilateral testicular pain
* Fever may also be a presenting feature in males.
Q.48. A 23 year old homeless smoker has an ongoing productive cough. She takes recreational drugs and looks malnourished. She has lost 15 kg in the past year. There are several nontender swellings on both sides of her neck. On examination, she has crackles in her right upper lobe but is otherwise well and stable. A chest X-ray reveals upper lobe infiltrates with cavitation on the right lung. What is the most likely diagnosis?
Correct Answer : C
It is important to remember the risk factors of tuberculosis as well as how it presents. Weight loss is seen in tuberculosis. One of the signs of tuberculosis is lymphadenopathy particularly cervical which is seen in this stem.
The examination findings and chest X-ray are consistent with almost every option given in the question. But only tuberculosis and lung cancer would have the associated weight loss. Given the age of only 23, tuberculosis is the likely answer.
Risk factors for TB which are included in this stem are:
• Homeless
• Drug abuser
• Smoker
The reason for homeless and drug abusers being at risk of TB is that they often live in overcrowded areas and have poor housing which encourages the spread of TB.
TUBERCULOSIS (TB) is an infection caused by Mycobacterium tuberculosis that most commonly affects the lungs.
Primary tuberculosis :
• Primary infection of the lungs
• A small lung lesion known as a Ghon focus develops
• Primary infection is usually asymptomatic
Secondary (post-primary) tuberculosis :
• Occurs if the host becomes immunocompromised the initial infection may become reactivated
• Presentation of secondary infection is variable and often nonspecific
Presentation :
• Fatigue, malaise
• Chronic, productive cough
• Night sweats
• Weight loss
Diagnosis :
• X-ray The upper lobe infiltrates with cavitation
• Stain for Acid-Fast Bacilli (AFB)
• Culture is most specific but may take weeks
• Nucleic acid amplification test. Only performed as an additional test in specific scenarios such as if the patient has HIV, or rapid information about mycobacterial species would alter the management. Do not use the Mantoux test to diagnose acute cases of TB Screening
• Mantoux testing to diagnose latent tuberculosis infection.
• Interferon Gamma testing : Used in patients who have had a bacillus Calmette-Guérin (BCG) vaccine. This is because the Mantoux test may be positive in patients who have had a BCG vaccine.
Management of active tuberculosis :
• Initial phase - first 2 months. Mnemonic (RIPE): Rifampicin, Isoniazid, Pyrazinamide, Ethambutol
• Continuation phase - next 4 months. Rifampicin, Isoniazid
Q.49. A 33 year old man comes from India with cough, fever and enlarged cervical lymph nodes. Histology reveals caseating granulomas found in the lymph nodes. What is the most likely diagnosis?
Correct Answer : A
Tuberculous lymphadenitis (or tuberculous adenitis) is a chronic specific granulomatous inflammation of the lymph node with caseation necrosis, caused by infection with Mycobacterium tuberculosis or Mycobacterium bovis.
Tuberculosis is responsible for up to 43 percent of peripheral lymphadenopathy in the developing world. In rural India, the prevalence of tuberculous lymphadenitis is significantly higher.
Q.50. A 33 year old man with Hodgkin’s lymphoma has chemotherapy 9 days ago. He develops a temperature of 39.0 C and signs of a chest infection. Blood count shows: Haemoglobin 113 g/L White cell count 2.3 x 109/L Neutrophils 0.8 x 109/L Platelets 150 x 109/L What is the most likely management?
Correct Answer : C
Piperacillin-Tazobactam may be used in the management of neutropenic patients with fever suspected to be due to a bacterial infection as in patients with post-chemotherapy neutropenia.
Q.51. A 4 year old girl is brought by her mother into the Paediatric Accidents & Emergency with a history of fever, headache and vomiting for the past 2 days. Her mother records her fever at home and says that her fever has not gone below 39 C for the past 24 hours. She is drowsy and difficult to wake up. She had a seizure causing twitching of the right side of the body at home for 4 minutes an hour ago. Her respiratory rate is 40 beats/minute, oxygen saturation is 90%, temperature is 39.2 C, and capillary refill time is 3 seconds. Her Glasgow Coma Scale is recorded as 11. During examination, she tends to turn away from any source of bright light. What is the most appropriate investigation that would lead to a diagnosis?
Correct Answer : D
The fever, headache, vomiting, and drowsiness are nonspecific but given there is absent history of symptoms of a urinary tract infection or respiratory infection, we should consider CNS involvement. The key phrase here is also photophobia which makes meningitis the top of your list of which a lumbar puncture is the method of choice for diagnosis.
For exam, a good trick which works most of the time is if the patient has got a rash, then perform blood culture as the diagnosis is most likely meningococcal septicaemia.
The causative organism is Neisseria meningitides. If there is no rash then a lumbar puncture (LP) would be the answer, but this can only be done if there are no signs of raised intracranial pressure (ICP).
Lumbar puncture contraindications to look out for during the exam:
• Glasgow Coma Scale less than 9 or a drop of 3 points or more
• Papilloedema
• Unequal, dilated or poorly responsive pupils
• Bulging tense fontanelle.
Q.52. A 24 year old man develops itching worse at night and following a bath. Examination reveals a greyish white linear rash that can be seen on the flexor surface of the wrist and axillary folds. What is the most likely diagnosis?
Correct Answer : B
Do not get distracted by the itch that is worse at night following a bath. Although we see that commonly in polycythemia, the greyish-white linear rash on the wrist points towards scabies.
Q.53. A 17 year old man has acute pain and earache on the right side of his face. He has a temperature of 39.4 C and has extensive pre-auricular swelling that is tender on palpation bilaterally. He also complains of headache, malaise, and dry mouth. What is the most likely diagnosis?
Correct Answer : D
Please see Q-40
Q.54. A 33 year old African woman presents with episodes of fever with rigors and chills for the past year. Blood film shows ring form plasmodium with Schuffner’s dots in red blood cells. What is the most appropriate drug to eradicate this infection?
Correct Answer : B
Schuffner’s dots are exclusively found in Plasmodium ovale and Plasmodium vivax. Thus primaquine should be used to eradicate them. The fact that they gave you the ethnicity, is another clue. Plasmodium ovale typically comes from Africa.
Q.55. A drug addict has bitten a police officer. The police officer attends the Accident and Emergency alone. He had a tetanus booster injection 3 years ago after cutting his hand on glass while gardening. On examination, there is broken skin with blood. What would you advise the police officer?
Correct Answer : C
The patient is at risk of acquiring a blood-borne viral infection such as hepatitis B, hepatitis C, and HIV. The assaulter is known to be a drug addict and therefore is at a higher risk of being infected with a blood-borne virus. This wound is considered high-risk, and post-exposure prophylaxis (PEP) should be started.
Testing for hepatitis B, hepatitis C, and HIV initially is recommended as a baseline test. This gives us clues to whether the police officer is already positive for HIV in which case the management differs. But the more important action is to start PEP as this can significantly change the outcome of the incident.
The police officer will also require follow-up serology blood tests at 6 weeks, 3 months, and 6 months.
All human bites should be treated with a 7-day course of oral co-amoxiclav.
Q.56. A 28 year old man presents to Accident & Emergency with the complaint of coughing up blood. He coughed up about a teaspoon of fresh blood this morning and he coughed up a few drops on his way to the hospital. He has had no other episodes of haemoptysis in the past. Upon further questioning, it is revealed that he has had a chronic cough for the past two months. He has no history of recent travel outside. He drinks alcohol socially and smokes around four cigarettes a day. He admits to smoking marijuana occasionally with his friends. The patient works as a packer in a busy loading bay for a large manufacturing company. He is extremely nervous and thinks that he might have cancer as he read on the internet that one of the symptoms of cancer is the coughing up of blood. Upon physical examination, the patient looks unwell and cachectic. Dullness to percussion can be appreciated over the right upper zone of his chest. A chest X-ray reveals right upper lobe consolidation with a bilateral pleural effusion. What is the most appropriate investigation for this patient?
Correct Answer : A
Clues that point to tuberculosis in this stem are cachexic, chronic cough, and hemoptysis. The chest X-ray findings are also consistent with tuberculosis. In tuberculosis, the chest X-rays usually show upper lobe infiltrates with cavitation. Although marijuana is in the stem, smoking marijuana is not an independent risk factor for developing tuberculosis.
The patient would be asked to produce 3 samples of a deep cough sputum (with at least 1 early morning sample) and these are sent away for acid-fast staining and culture. Pleural aspiration as a diagnostic procedure is usually performed in unilateral pleural effusions where there is diagnostic uncertainty. Pleural aspiration is not routinely used as a diagnostic procedure for tuberculosis although it can detect TB.
Q.57. A 38 year old woman presents with dysphagia and a lump on her neck. On examination, there is a 5 cm by 4 cm erythematous neck swelling lateral to the thyroid cartilage. She has a temperature of 38.9 C, a respiratory rate of 28 breaths/minute and a heart rate of 110 beats/minute. What is the most appropriate action?
Correct Answer : D
Superficial neck abscesses are usually the result of an infection in a lymph node in the neck (lymphadenitis) turning into an abscess. Staphylococcus aureus is usually the culprit.
Antibiotics can initially be given and if the abscess does not resolve then it may need to be drained. However, given this stem, it is clear that she is unwell and possibly septic, and thus oral antibiotics would not be a good option here. Antibiotics need to be given intravenously, and the abscess needs to be drained.
Most cases of neck abscess require patients to be hospitalized and intravenous antibiotics to be given. Analgesia is also important to relieve pain from the abscess but the option to give oral antibiotics in combination with pain relief is an incorrect one in a scenario where the patient is potentially septic.
Draining of a superficial neck abscess is a simple procedure and is done under general anesthesia. Incision and drainage are important as the pus sample collected from the drainage is sent to the laboratory for culture and sensitivity. After the organism is identified, specific antibiotics can be used accordingly.
Q.58. A 38 year old female, 32 weeks pregnant presents with thick white marks on the inside of her mouth for 3 weeks. Her mouth including her tongue appears inflamed on examination. She smokes 20 cigarettes a day despite advice to quit. What is the most likely diagnosis?
Correct Answer : B
Smokers are more likely to develop oral thrush. The history of pregnancy is not too relevant. But the idea that the question writers want to portray is that in pregnancy, the immune system is weakened. Thus candidiasis is more likely. Lichen planus may have a lace-like appearance and not a thick white mark like in this case.
Aphthous ulcers are typically round or oval sores or ulcers inside the mouth. Molluscum contagiosum is present as firm, smooth, umbilicated papules on the trunk or extremities and not in the mouth like in the given stem. Leukoplakia is an option but it is less likely that oral candidiasis. They may sometimes give a history of a white lesion in the mouth that cannot be rubbed off.
Q.59. A 22 year old lady comes to the hospital with complaints of fever, vertigo and pain in her right ear. On physical examination, there are vesicles at the right external auditory meatus. What is the most likely diagnosis?
Correct Answer : A
Please see Q-10
Q.60. A 32 year old breast feeding mother has a paniful, swollen, hard lump in her right breast which developed 2 weeks after having a normal vaginal delivery. She has a temperature of 38.1 C. A breast abscess is suspected. What is the most likely organism?
Correct Answer : C
Infectious mastitis occurs when accumulated milk allows bacteria to grow. It typically develops within the first few weeks of breastfeeding. If treatment is delayed it may result in a breast abscess like in this case. The usual infecting organism is Staphylococcus aureus, although it may also be caused by Staphylococcus albus and streptococci.
If a breast abscess forms, advise the patient to discard the milk if it is pus-like. Flucloxacillin is usually the treatment of choice. It is safe for the baby. Incision and drainage of abscess with cavity packed open with gauze is recommended if the underlying skin is thin or necrotic.
If the patient has mastitis, we would still encourage the patient to breastfeed as the blocked milk ducts may clear more quickly by breastfeeding as it keeps milk flowing. However, if the patient has an abscess, in some cases, breastfeeding may have to cease until the abscess is successfully treated.
Q.61. A 70 year old man presents to the Emergency Department with fatigue, fever and a productive cough. He is currently on his 4th course of chemotherapy for small cell carcinoma of his lungs of which finished 8 days ago. On auscultation, there are coarse crepitations on his right lung base. He has a temperature of 37.6 C, respiratory rate of 17 breaths/minute, pulse of 85 beats/minute and a blood pressure of 120/80 mmHg. His blood results come back showing the following: Haemoglobin 122 g/L White cell count 1.1 x 109/L Platelets 90 x 109/L What is the most appropriate action?
Correct Answer : B
This stem should ring bells of neutropenic sepsis. The question deliberately left out the count for neutrophils. This is usually reported together with the white cell counts as it is part of the request for a full blood count. Nonetheless, the white cell count is shown to be low here which is consistent with neutropenic sepsis.
Neutropenic sepsis should not be taken lightly. They can deteriorate rapidly and become hypotensive within hours. If the patient has received chemotherapy within the last 4 weeks and is now having a fever and feeling unwell, he should immediately be started on intravenous broad-spectrum antibiotics. This should ideally be started even before the results of the full blood count come back.
Q.62. A 69 year old woman lives in a nursing home following a stroke. She recently developed a reddish scaly rash on her trunk. She has many scratch marks on her limbs and trunk with linear burrows seen on her hands and feet. What is the most appropriate treatment?
Correct Answer : B
Please see Q- 12
Q.63. A 67 year old man who is currently on chemotherapy treatment for a malignancy suddenly develops febrile neutropenia. He has been commenced on tazocin and gentamicin. He has recently commenced meropenem but his fever still remains at 39 C on the 4th day. Two blood tests and urine cultures have come back negative. Investigation done this morning show: Haemoglobin 104 g/L White cell count 0.5 x 109/L Platelets 35 x 109/L What is the best management?
Correct Answer : B
Continue inpatient empiric antibiotic therapy in all patients who have unresponsive fever unless an alternative cause of fever is likely. It is quite the norm to add on IV antifungals for neutropenic sepsis as well. If there was an option to order investigations for fungal infections in this question, that would be correct as well.
General Management for Neutropenic Sepsis :
• Antibiotics must be started immediately (do not wait for the WBC)
• Start empirical antibiotic therapy with piperacillin with tazobactam (Tazocin) immediately
• If the patient is still febrile and unwell after 48 hours, an alternative antibiotic such as meropenem is often prescribed +/- vancomycin
• If the patient is not responding after 4-6 days order investigations for fungal infections, rather than just starting antifungal therapy blindly.
Q.64. A 30 year old man returned from a business trip to Indonesia 6 days ago presenting with complaints of fever, headache, vomiting, joint and muscle ache. His headache is felt behind the eyes and has been present for the past 2 days. What is the most likely diagnosis?
Correct Answer : C
Dengue is characterized by an abrupt onset of fever often accompanied by severe headache and retro-orbital pain, myalgia, arthralgia, nausea, vomiting, and abdominal pain Retro-orbital pain is a well-recognized feature of dengue fever.
Q.65. A 46 year old man is being investigated for indigestion. A jejunal biopsy shows deposition of macrophages in the lamina propria-containing granules which stain positive for Period Acid-Schiff (PAS). What is the most likely diagnosis?
Correct Answer : D
A jejunal biopsy shows deposition of macrophages in the lamina propria-containing granules which stain positive for Periodic Acid-Schiff (PAS) and are diagnostic for Whipple’s disease.
Whipple’s disease is a rare disease featuring GI malabsorption which usually occurs in middle-aged white males. It is fatal if untreated and is caused by Tropheryma whipped, which, combined with defective cell-mediated immunity, produces a systemic disease.
Features:
• Often starts insidiously with arthralgia (mainly peripheral joints)
• GI symptoms commonly include colicky abdominal pain, weight loss, steatorrhea/diarrhoea, which leads to malabsorption.
• Systemic symptoms such as chronic cough, fever, sweats, lymphadenopathy, and skin hyperpigmentation also occur
• Cardiac involvement may lead to endocarditis
• CNS features include reversible dementia, ophthalmoplegia, and facial myoclonus.
• Diagnosis requires a high level of clinical suspicion
• Jejunal biopsy shows stunted villi. There is deposition of macrophages in the lamina propria-containing granules which stain positive for Periodic Acid-Schiff (PAS).
Q.66. A 40 year old man who recently traveled to Sudan 5 weeks ago presents with dark urine, rigors and a fever. On examination, a tender hepatomegaly was noted. What is the most likely diagnosis?
Correct Answer : B
This is a very interesting question with a huge debate between Malaria and Schistosomiasis. Initially, one may pick Schistosomiasis as Schistosomiasis is widely distributed in Sudan. 5 weeks here fits the timeline of schistosomiasis as symptoms usually take from four to six weeks from the time of infection.
They may feel generally unwell at this point however in the majority of cases blood in the urine only occurs somewhere between 10 to 12 weeks after the infection. The blood in urine is due to the worms of Schistosoma haematobium migrating to the veins around the bladder and ureters.
In short, a patient suffering from urinary schistosomiasis (caused by Schistosoma haematobium) will develop terminal haematuria, while another patient infected with Schistosoma mansoni may develop hepatomegaly but ultimately these symptoms are caused by different species. Considering the clinical features, the best answer is Malaria.
Blackwater fever is a complication of malaria infection in which red blood cells burst in the bloodstream (hemolysis), releasing hemoglobin directly into the blood vessels and the urine. This accounts for the dark urine. Malaria is also known to cause hepatomegaly.
Q.67. A 16 year old girl has had an enlarging mass in the right side of her neck for the past 2 weeks with a sore throat. She feels tired and weak. She has several smaller associated lymph nodes that are palpable at her axilla. Oropharyngeal examination shows tonsillar enlargement which is exudative. What is the most likely diagnosis?
Correct Answer : A
Please see Q-18
Q.68. A 28 year old man presents with a deep penetrating wound on his foot after having stepped on a nail in a field. He said he had the full course of tetanus vaccine when he was in school and again at 18 years old. What is the most appropriate management to be given?
Correct Answer : C
If the patient has received a full five-dose course of tetanus vaccine at the recommended intervals or is up-to-date with their tetanus immunization schedule, no further doses of vaccine are recommended even with a tetanus-prone wound however, high-risk wounds still need intramuscular human tetanus immunoglobulins.
There are some exceptions when tetanus boosters are indicated for patients who have completed their five-dose course of tetanus vaccines but are traveling to remote areas, but this is unlikely to be asked during the exam.
Q.69. A 33 year old woman has a 2 day history of numerous painful ulcers on her vulva. She is sexually active and has multiple partners in the past. What is the most likely cause of her ulcers?
Correct Answer : B
The probable diagnosis here given the symptoms of numerous painful ulcers is genital herpes which is caused by HSV type 2. The two distractors here are Treponema pallidum and Varicella-zoster virus.
Treponema pallidum which causes syphilis presents with PAINLESS papules that later forms PAINLESS ulcers.
Varicella-zoster causes shingles if reactivation occurs later in life which presents with vesiculation and ulceration in a dermatomal distribution.
Q.70. A 30 year old woman had unprotected sexual intercourse 48 hours ago with a man who is HIV positive and takes antiretroviral therapy. His last viral load a week ago was 1000 copies/mL. What is the most appropriate action?
Correct Answer : D
Post-exposure prophylaxis (PEP) should always be given for 28 days. Serological testing immediately is appropriate as a baseline test.
Drugs used in PEP are a combination of oral antiretrovirals to be given as soon as possible for 28 days. This reduces the risk of contracting HIV.
Q.71. A 3 year old boy presents with a two day history of being unwell. He has a two hour old rash made up of urticarial and purpuric spots. His level of consciousness is decreased. He has a blood presssure of 80/50 mmHg, a respiratory rate of 30 breaths/minute, oxygen saturation of 94% on room air, a temperature of 39 C and a capillary refill time of 3 seconds. A urine dipstick was found to be unremarkable. What is the investigation most likely to lead to a diagnosis?
Correct Answer : B
A very straightforward question. The child is septic with the likely diagnosis of meningococcal septicaemia. Blood culture is the investigation of choice.
Q.72. A 35 year old woman has numerous painful blisters and sores on her vulva. She also complains of a flu-like illness with mild fever starting a few days ago. The pain in the vulva is so severe that she refuses to go to the toilet to urinate. She is prescribed pain relief. What other treatment should be offered?
Correct Answer : C
The diagnosis here is genital herpes. The pain caused by genital herpes is so severe that it can cause urinary retention. Genital herpes is treated with acyclovir.
Q.73. A 68 year old woman has a sudden onset of pain and loss of hearing in her left ear and unsteadiness when walking. There are small lesions visible on her palate and left external auditory meatus. What is the most likely diagnosis?
Correct Answer : A
Please see Q-10
Q.74. A 3 year old boy going to nursery has developed chicken pox. What advice will you give the parents?
Correct Answer : B
The most infectious period is 1-2 days before the rash appears, but infectivity continues until all the lesions are dry and have crusted over (usually about 5 days after the onset of the rash). Therefore, children with chickenpox should be kept away from school or nursery until all the vesicles have crusted over.
Q.75. A 14 year old boy has pain and swelling at the angles of the jaw bilaterally. He has a temperature of 38.4 C. He has been complaining of dry mouth and sore ears and he finds it difficult to talk. On examination, his scrotum is also swollen and oedematous and the testes are impalpable. What is the most likely diagnosis?
Correct Answer : C
Please see Q-40
Q.76. A 7 year old boy presented 10 hours after having a foot injury while playing football in the garden. A metal spike had gone through his shoes and pierced the bottom of his foot. His immunisations are up to date. What is the most appropriate management?
Correct Answer : A
This injury is to be considered a high-risk wound. Intramuscular human tetanus immunoglobulin needs to be administered. Antibiotics would also be useful to add here to prevent wound infection.
Q.77. A 29 year old lady with a history of intravenous drug use was investigated for symptoms of malaise and loss of appetite. Blood tests showed: HBsAg negative Anti-HBC positive Anti-HBs negative HCV antibody reactive HCV RNA detected In addition to this, liver biopsy produced a histopathological sample that demonstrated moderate necrosis and inflammation with definite steatosis. What is the most likely cause for these findings?
Correct Answer : C
Current Hepatitis C virus infection is the correct answer given that the serology is indicative of likely resolved hepatitis B infection as well as the presence of HCV RNA. The histopathological findings in this case are helpful to you if you can remember this from your early days in medical school, however, the serology is enough to get the answer. So don’t be thrown by details like this. Given that the patient is an intravenous drug user, you should consider blood-borne viruses.
Remember hepatitis C is associated with cirrhosis and eventual hepatocellular carcinoma.
Past Cytomegalovirus infection is not correct as there is no indication in the stem or the results that this infection is present. Current Human Immunodeficiency Virus infection is not correct as there is no indication in the stem regarding HIV infection although it is a blood-borne virus. Therefore it is there to try to confuse you.
Previous Hepatitis E virus infection is not the correct answer as there is no mention of hepatitis E virus in the results and this is usually transmitted via faecal-oral route in less developed countries. There is no indication of this in the question.
Current Hepatitis B virus infection is not the correct answer as the blood test results show an equivocal test result which is commonly due to resolved infection.
This is very probable in this case as this is a blood-borne infection and this patient has a major risk factor for contracting this in her life. A relatively low proportion of people with hepatitis B progress to chronic infection.
Q.78. A 58 year old woman attends clinic for advice as her grandson who lives with her has developed chicken pox with the rash appearing 2 days ago. She is currently undergoing chemotherapy for breast cancer and has been using long term corticosteroids to manage her inflammatory bowel disease. She has never had chicken pox before. On examination, there is no evidence of any rash. What is the most appropriate management?
Correct Answer : B
The patient is immunocompromised. She is on chemotherapy AND she is taking long-term steroids. She is at a high risk of developing severe complications from varicella infection. As she is currently asymptomatic, the best management would be immunoglobulins. If she had developed the infection, then we would administer aciclovir.
Q.79. A 65 year old man visited his local GP with complaints of difficulty and pain in swallowing. These symptoms have been occurring for approximately two weeks now. His appetite has also decreased and he has noticed some white patches on his tongue. His medical history is significant for non-insulin dependent diabetes mellitus, diagnosed two years ago. He claims that his condition is well controlled and that he is compliant with his medication. He also casually mentions that he quit smoking six years ago. During the consult, the patient appears to be in a generally good state of health. He reported a recent use of antibiotics for an upper respiratory tract infection that he recovered from a week ago. On examination, the patient’s vital signs, including his blood glucose level, were all found to be within normal limits. Examination of the head and neck region revealed no palpable lymph nodes. An extra-oral examination performed proved to be insignificant. An intra-oral examination of the patient revealed an extensive white plaque on the midline of the posterior dorsal portion of the tongue measuring approximately 1.5 centimetres in diameter. What is the most likely treatment option for this patient?
Correct Answer : D
This patient is suffering from oral thrush which is also known as oral candidiasis. It is an infection caused by a yeast germ called Candida albicans. This patient has quite a few risk factors for oral thrush. Firstly, he is an elderly man who is diabetic and thus, immunocompromised. He also had a recent course of antibiotics which also contributes to the susceptibility of getting oral thrush.
The clincher to diagnose oral candidiasis in this patient is his positive examination findings. The patches generally progress to form larger plaques and the presence of candida may become painful and cause discomfort in eating food.
The most appropriate treatment option for this patient is an oral fluconazole suspension.
It helps to clear fungal infections in people with a poor immune response who develop extensive oral thrush.
Chlorhexidine mouthwash is an antibacterial mouthwash. It is mainly used to treat gingivitis and is used for the maintenance of oral hygiene. So, that is not the correct option.
Q.80. A 30 year old homeless lady has cough, sputum and a fever for the past few months. She complains of night sweats and has lost 13 kg in the past 6 months. A chest X-ray was performed which showed apical involvement with infiltrates and cavitation in the upper lobe of the right lung. What is the next most appropriate test to perform?
Correct Answer : B
It is important to remember the risk factors of tuberculosis as well as how it presents. Weight loss is common because of the chronicity of the infection. Night sweats also occur with TB. Homeless people are at higher risk of TB as they often live in overcrowded areas and have poor housing which encourages the spread of TB. It is estimated that the homeless are 150 times more at risk of TB.
Stain for Acid-Fast Bacilli (AFB) is the most appropriate next test.
Mantoux test and interferon-gamma test are used to diagnose latent tuberculosis infection and not acute cases of tuberculosis.
Bronchoscopy is sometimes used when there is a need to obtain bronchoalveolar lavage or lymph node samples. It is used if a patient has a non-productive cough or an unhelpful sputum culture but the physicians still have a high index of clinical suspicion for tuberculosis.
Q.81. A 36 year old homosexual man has multiple purple nodular lesions on his face and upper trunk measuring 1-2 cm across. It is not painful or itchy. What is the most likely diagnosis?
Correct Answer : A
Please see Q-30
Q.82. A 30 year old man is seen in the emergency department with vomiting, muscle pain, rash at his axilla and sensitivity to light. He has a temperature of 38.9 C. The medical staff are suspecting he is suffering from meningitis. Which is the most appropriate empirical antibiotic to be started immediately?
Correct Answer : C
In a hospital setting, give intravenous third-generation cephalosporin antibiotics.
If these same questions were given but she presented them to her GP (or was found outside the hospital), then benzylpenicillin IM or IV would be the correct answer.
If you suspect meningitis and the patient is not yet in the hospital give IM/IV benzylpenicillin and send the patient to the hospital.
If this same question was given, but there was a diagnosis of Listeria, then IV amoxicillin and gentamicin would be the correct answer.
If this same question was given, but she is penicillin or cephalosporin allergic, then chloramphenicol would be the answer.
Generally, treatment for meningitis should be started before doing any investigations due to the seriousness of the disease.
If the patient has a rash, then perform a blood culture as the diagnosis is most likely meningococcal septicemia. The causative organism is Neisseria meningitides.
Q.83. A 30 year old man presents to the emergency department with difficulty breathing. He has returned from India 5 days ago. On examination, his throat reveals grey membranes on the tonsils and uvula. He has fever and enlarged anterior cervical lymph nodes. What is the most likely diagnosis?
Correct Answer : A
History of travel to India with grey membrane in tonsil and uvula, and a low-grade fever, supports the diagnosis of diphtheria.
Note that the pseudomembrane may cause respiratory obstruction as seen in this question.
Q.84. A 21 year old man has generalized skin lesions. The skin lesions consist of macular, papular and vesicles and concentrated more on his back and chest. Pinkish fluid is seen secreted from a few of the lesions. He has a temperatutre of 39.1 C. What is the most appropriate medication to prescribe?
Correct Answer : B
The skin lesions seen in different stages are quite classic for chicken pox. As this man has a temperature of 39.1°C, there is likely a secondary infection that requires oral antibiotics.
Usually, cases of chicken pox only require symptomatic treatment but if chicken pox blisters become infected with bacteria, they would require antibiotic treatment. With a temperature of 39.1°C, oral antibiotics is more suitable than topical.
Q.85. A 24 year old college student presents to A&E with nausea, vomiting, headache, neck stiffness and a fever of 38.4 C. What is the most appropriate empirical antibiotic to be started immediately?
Correct Answer : D
She has signs of meningitis. Intravenous Ceftriaxone needs to be administered immediately.
Q.86. A 3 year old child was treated for bacterial meningitis and has recovered from it. She is now afebrile. What is the most appropriate investigation to perform?
Correct Answer : A
One of the delayed complications of meningitis is decreased hearing, or deafness. Hearing loss may be partial or total. People who have had meningitis will usually have a hearing test after a few weeks to check for any problems.
Q.87. A 23 year old homosexual man presents to the genitourinary medicine clinic with urethral discharge. A few weeks ago, he had a painless ulcer on his penis which healed. He is sexually active. What is the most appropriate investigation to confirm the diagnosis?
Correct Answer : C
This man has had a painless ulcer (chancre) which healed. This is the primary stage of syphilis.
The reason the stem included that he is homosexual is that syphilis is more common in men who have sex with men.
If syphilis is suspected, blood for VDRL, TPHA, or treponemal antibody absorption should be sent. These are serology tests.
Polymerase chain reaction (PCR) or dark field microscopy can only be carried out from a sample of the lesion or an infected lymph node. Since the ulcer is healed, a sample cannot be obtained. If there was an ulcer visible, a dark field microscopy would allow a point-of-care syphilis diagnosis.
Q.88. A 20 year old man with a known diagnosis of otitis media presents with a severe headache, and sensitivity to light. He is shivering, sweating and has a temperature of 38.9 C. What is the most likely complication?
Correct Answer : B
One of the very serious but rare complications of otitis media is meningitis. This patient has clear signs that resemble meningitis.
Q.89. A 44 year old man with rheumatoid arthritis has come to clinic with concerns as his partner who he has lived with has developed shingles 2 days ago. He has been taking long term prednisolone and methotrexate to manage his rheumatoid arthritis. He gives a history that he has had chicken pox when he was 5 years old. On examination, there are no lesions noted on his skin. What is the most appropriate action?
Correct Answer : B
This patient is immunocompromised. The management for immunocompromised patients exposed to chicken pox differs greatly.
Some physicians may consider offering oral aciclovir to the immunocompromised when exposure is significant.
People who have had a significant exposure to chickenpox and who are immunocompromised should be tested for varicella zoster antibody, regardless of their history of chickenpox.
Q.90. A 7 year old child is being investigated for active respiratory tuberculosis. He has dry cough and is unable to produce sputum. His parents have been informed about the possibility of a bronchoalveolar lavage which can be useful in diagnosing tuberculosis however the parents decide this invasive test. What is the next method to acquire a sample to diagnose tuberculosis?
Correct Answer : C
The diagnosis of tuberculosis is usually made in one of three ways:
1. Smear of sputum - Staining with Ziehl-Neelsen (ZN) stain and microscopy for acid-fast bacilli
2. Culture of sputum - Takes 4-8 weeks due to slow bacterial growth
3. Histology with caseating granulomas on biopsy
This young child is unable to cough up sputum and thus there is no sample to perform a stain for Acid-Fast Bacilli (AFB).
Bronchoalveolar lavage would be the next step to attempt to acquire a sample given the high index of suspicion for tuberculosis.
As the parents of the child have declined this procedure, gastric lavage would be able to acquire a sample. Some patients with tuberculosis do not raise any sputum but instead swallow small amounts of sputum.
Q.91. A surgeon was pricked by a needle used during an appendectomy. Basic first aid for the surgeon has been performed. After the operation, the patient was informed of the incident and gave her consent to be tested for blood-borne infections for the benefit of the surgeon. The patient states that she is a dental nurse, practices safe sexual intercourse and denies use of any intravenous drugs. She has never been tested for blood-borne infections. The surgeon remembers being immunized for hepatitis B but cannot remember the last time he received a booster for hepatitis B. What is the most appropriate action for the patient and the surgeon on the day of the incident?
Correct Answer : C
Basic first aid for needle stick injuries includes washing with soap under running water and encouraging bleeding in that area. When there is a needle stick injury to health care professionals, occupational health must be contacted. The patient should be informed that there has been a needle stick injury and permission should be asked before taking blood for HIV antibodies, hepatitis B surface antigen, and hepatitis C virus antibodies.
Q.92. A 19 year old woman attends the genitourinary medicine clinic with a few cauliflower-like growths of varying size at the vulva after returning from Spain several weeks ago. She is concerned and would like treatment. She has no previous medical conditions. What is the most appropriate treatment?
Correct Answer : A
Genital warts are benign epithelial skin tumors commonly seen as cauliflower-like growths. They are most commonly caused by HPV types 6 and 11. Transmission is through sexual intercourse. The incubation period is between 3 weeks to several months.
Gardasil is a quadrivalent vaccine that protects against 4 types of HPV: 6, 11, 16, and 18.
HPV 6 and 11 are responsible for benign genital warts, and HPV 16 and 18 are responsible for most cervical cancers. Gardasil does not treat established genital warts.
Treatment options include podophyllotoxin, imiquimod, and ablative methods such as cryotherapy and excision under local anesthetic.
Q.93. A 44 year old woman with HIV attends the HIV clinic. Her last blood test shows her CD4 count to be at 160 cells/mm3. Her CD4 count has been persistently below 200 cells/mm3. The clinician suggests starting prophylactic antibiotics to prevent pneumocystis carinii pneumonia. What is the most appropriate antibiotic?
Correct Answer : C
Its recommended to use co-trimoxazole as pneumocystis carinii pneumonia.
Prophylactic treatment of choice for HIV-seropositive patients with persistent CD4 counts less than 200 cells/mm3. Azithromycin has a role in prophylaxis against Mycobacterium avium complex in patients with HIV. It is used when CD4 counts fall below 50 cells/mm3.
Q.94. A man with suspected active TB wants to be treated at home. What should be done to prevent the spread of disease?
Correct Answer : D
Universal prevention application protocol.
Q.95. A 47-year-old man has a temperature of 39C and is delirious. He has developed blisters mainly on his trunk, which appeared a few hours ago. He is well and not on any medications. He last travelled 5 months ago to Italy. Which of the following is the most likely diagnosis?
Correct Answer : B
Centripetal distribution of blisters favours chickenpox. Adults more commonly develop a more generalized brain inflammation ("encephalitis") whose symptoms may include delirium and seizures. The incubation period of chicken pox is 10-21 days. So this travel history is not significant.
Q.96. A 33-year-old man presents with an itchy scaly annular rash on his thigh after a walk in the park. Which of the following drugs will treat his condition?
Correct Answer : B
Itchy scaly annular rash after a walk in the park indicates erythema migrans caused by the spirochete Borrelia Burgdorferi transmitted by bite of pinhead-sized Ixodes ticks leading to Lyme disease.
Q.97. A middle aged asian man presents with episodes of fever with rigors and chills for last 1 year. Blood film: ring form plasmodium with schuffner’s dots in RBCs. What is the drug to eradicate this infection?
Correct Answer : A
Shuffner dots inside RBC indicate, it is plasmodium ovale or plasmodium vivex infestation. Eradication of the disease needs eradication of the liver cycle. Only primaquine is the drug that can eradicate the hepatic or liver cycle.
Q.98. A 30-year-old man presents with deep penetrating knife wound. He said he had TT when he left school. What will you do for him now?
Correct Answer : D
The wound is a tetanus prone wound. Here the man has received an adequate priming course of the tetanus vaccine but the last dose was taken more than 10 years ago. So he should take an immediate reinforcing dose of vaccine.
Q.99. A patient who came from India presents with cough, fever and enlarged cervical lymph nodes. Examination: caseating granulomata found in lymph nodes. What is the most appropriate diagnosis?
Correct Answer : B
Cough, fever, and cervical lymphadenopathy following travel to India makes the likely diagnosis of TB. Also caseating granulomata are diagnostic of TB.
Q.100. A 76-year-old woman has become tired and confused following an influenza like illness. She is also breathless with signs of consolidation of the left lung base. What is the most likely diagnosis?
Correct Answer : C
Infection toxicity or toxic shock syndrome is likely here as preceding flue like illness points towards current staphylococcal pneumonia which resulted release of enterotoxin type B leading to infection toxicity.
Q.101. A 55-year-old man has a chronic cough and sputum, night sweats and weight loss. What is the most likely causative organism?
Correct Answer : D
Chronic cough and sputum, night sweats and weight loss are classic features of tuberculosis.
Q.102. A 30-year-old man presents to the emergency department with difficulty breathing. He has returned from India. Examinations: throat reveals grey membranes on the tonsils and uvula. He has mild pyrexia. What is the most relevant diagnosis?
Correct Answer : A
Diphtheria is a serious infection caused by strains of bacteria called Corynebacterium diphtheriae that make toxins (poison). History of travel to India, grey membrane in tonsil and uvula, low-grade fever, and dyspnoea suggest the diagnosis of diphtheria.
Q.103. A 20-year-old lady is suffering from fever and loss of appetite. She has been diagnosed with toxoplasmosis. What is the treatment?
Correct Answer : B
If the eye is involved, or if immunocompromized, the treatment option is pyrimethamine + sulfadiazine.
Q.104. A 34-year-old man from Zimbabwe is admitted with abdominal pain to the emergency department. An abdominal X-ray reveals bladder calcification. What is the most likely cause?
Correct Answer : D
Bladder involvement is caused by Schistosoma hematobium while Schistosoma mansoni causes intestinal disease.
Q.105. 28-year-old man presents with a maculopapular rash over his trunk and palms. He also has numerous mouth ulcers. He had a penile ulcer which healed 2wks ago. What will you do to confirm the diagnosis?
Correct Answer : A
Nontreponemal antibody test if positive indicates that there may be syphilis and it is not confirmatory alone.
That is why a treponemal antibody test should also be done to confirm it. On the other hand resolved disease may show a negative treponemal test which is confirmed by a positive non-treponemal test.
Q.106. A 34-year-old man was walking along the country side when an insect bit him. After which he started to complain of an annular rash spreading upwards. What is the treatment you will give him?
Correct Answer : B
Described rash is erythema migrans which is a characteristic skin lesion seen in Lyme disease.
Lyme disease is caused by the bacteria Borrelia burgdorferi which is spread by ticks. The most common sign of infection is an expanding area of redness on the skin, known as erythema migrans, that appears at the site of the tick bite about a week after it occurred. Lyme disease is treated with Doxycycline.
Q.107. A 67-year-old man being managed for a malignancy develops neutropenic fever. He has been commenced on Ticarcillin, Tazobactam and Gentamicin. He has also recently commenced on Meropenem but on the 3rd day his tempeature still remains >39C. 2 blood tests and urine cultures show no organism. Investigations: Hgb 10.4 g/dl, WBC <0.5, Platelets 15. What will you do next?
Correct Answer : B
If judicial antibiotics fail to control fever next step is to add antifungal agents.
Q.108. A 32-year-old man has been to Thailand and returned with cervical lymphadenopathy and fever. What is he most likely suffering from?
Correct Answer : B
Cervical lymphadenopathy mentioned in question which occurs in EBV infection.
In Acute retroviral syndrome or primary HIV infection, there is generalized lymphadenopathy.
Infectious mononucleosis, or mono, refers to a group of symptoms such as fever, sore throat, swollen lymph glands in the neck and armpits, headache, fatigue, muscle weakness, swollen tonsils, night sweats usually caused by the Epstein-Barr virus (EBV). It typically occurs in teenagers, but you can get it at any age. The virus is spread through saliva, which is why some people call it “the kissing disease.”
Q.109. A 44-year-old man went on holiday to Sudan 5 weeks ago. He now presents with red urine and fever. Examination: Hepatomegaly. What is the most likely diagnosis?
Correct Answer : D
Holiday in Sudan, hematuria, fever and hepatomegaly suggest Schistosomiasis.
Q.110. What is the mode of spread of chicken pox?
Correct Answer : A
Mainly airborne but some cases may occur in close contacts.
Q.111. A 16 weeks pregnant patient who was exposed to a child with chicken pox came to GP for help. She was tested –ve for varicella antibody. What is the next most important step in management?
Correct Answer : B
If you are pregnant, come into contact with chickenpox case, found antibody negative on blood test you have to take injection Ig.
Q.112. A 5-year-old boy was brought to GP with high temperature and many vesicles on his back. What is the most appropriate management?
Correct Answer : D
A case of chickenpox. None of the given treatments is used in chickenpox. Symptomatic treatments like acetaminophen if fever, antihistamine, and calamine lotions are given.
Q.113. A 5-year-old child came from Ghana 6 weeks ago. 2 days ago he developed fever, vomiting and neck stiffness. He had taken malaria prophylaxis and had no rash. What is the diagnosis?
Correct Answer : B
The incubation period of malaria is 7 - 30 days. Malaria prophylaxis cannot give confirmed protection and there is often failure of prophylaxis.
Fever, vomiting, and neck stiffness are consistent with cerebral malaria. In meningococcal disease, there may be a rash. However meningococcal meningitis may be without rash. But Ghana is the endemic zone for malaria. So cerebral malaria is a stronger probability.
Q.114. A 25-year-old woman has a recent cough, hoarseness and swelling in the neck. There are several nontender swellings on both sides of her neck. She has lost 13kgs. She takes recreational drugs. What is the most probable diagnosis?
Correct Answer : D
Cough, wt. loss, non-tender cervical lymphadenopathy, and the immunodeficient state as a result of recreational drug use all these favours the diagnosis of TB. Laryngeal involvement may be the cause of horseness.
Q.115. A 45-year-old man has developed an annular rash with a scaly edge on his thigh. The rash has been spreading over the last 3weeks. He has some general aches and pains. What is the most useful investigation?
Correct Answer : C
Spreading annular rash suggests erythema migrans of Lyme disease.
Q.116. A 30-year-old man from Australia returned from a business trip to Indonesia 6 days ago presenting with complaints of fever, joint and muscle ache and headache, in particular behind the eye for the past 2 days. What is the most probable diagnosis?
Correct Answer : D
Fever, arthralgia, myalgia, and headache are common in dengue fever. Particularly retro-orbital pain is well well-recognized feature of dengue fever.
Q.117. A 25-year-old woman presents with a painful shallow ulcer on the vulva. What investigation has to be done?
Correct Answer : C
Herpes simplex virus causes multiple painful vesicles and syphilis ulcer is painless. As here single painful ulcer probable diagnosis is Chancroid caused by Hemophilus ducreyi.
Q.118. A 7-year-old child is being investigating for TB. His parents don’t agree for taking a BAL (broncho-alveolar lavage). what other sample will show about growth of the organism?
Correct Answer : C
Sputum can not be obtained in extremes of age as in too young or elderly. When initial dry cough of TB sputum may not be coughed up by patients as well.
So in such cases bronchoalveolar lavage (BAL) can demonstrate the bacteria. If BAL is refused alternate other procedures such as gastric washing can be done.
When a patient coughs the saliva is contaminated with coughed-up droplets and swallowed by the patient. So gastric lavage can bring these organisms with samples of gastric washing.
Q.119. A homeless lady presents with cough and fever. She complains of night sweats and weight loss. CXR has been done and shows opacity. What is the next appropriate management?
Correct Answer : A
Cough, fever, night sweats, and weight loss in a homeless lady are very suggestive of TB. So we should do AFB as her next investigation.
Q.120. A lady from Asia presented with lump in her neck. FNAC has been done and revealed lesions with caseous material in the center surrounded by fibrosis. What is the most probable diagnosis?
Correct Answer : B
Central caseous necrosis with surrounded fibrosis is diagnostic of TB lymphadenitis. The most common cellular components seen are epithelioid cell clusters, lymphocytes, and Langhans giant cells.
Q.121. An 18-year-old man has a smooth, tender swelling extending from the ear to the angle of the jaw of sudden onset. Temp 38.5C. What is the most likely diagnosis?
Correct Answer : B
Mumps is a viral disease caused by the mumps virus. Initial symptoms are non-specific and include fever, headache, malaise, muscle pain, and loss of appetite. These symptoms are usually followed by painful swelling of the parotid glands, called parotitis, which is the most common symptom of infection.
Q.122. A 34-year-old man from Asia presented with 5 months history of productive cough, night sweats and weight loss. His CXR reveals some shadowing in the left upper zone. What is the most discriminating investigation?
Correct Answer : A
Cough, night sweats, and weight loss are common features of TB. Apical lesions in the lung on chest X-ray are suggestive of PTB (pulmonary tuberculosis). The most discriminating investigation is AFB for sputum.
Q.123. A 10-year-old boy is brought to the emergency department 10 hours after injury to the foot. It was punctured with a metal spike that passed through his shoe. What is the next best step?
Correct Answer : D
“NEXT” best step is clean the wound.
Q.124. A 29-year-old man took a tour of Japan and also travelled to other parts of Asia, developed fever, petechie and rash on his body. He didn’t take malaria prophylaxis prior to travel. What is the most likely diagnosis?
Correct Answer : D
Dengue fever
Q.125. A 46-year-old male presents with confusion and drowsiness. What is the most likely diagnosis?
Correct Answer : A
This type of question can be regarded as cns infection in AIDS and some may argue there is no mention of AIDS. If you consider with immunocompetent patient likely answer may be HSV and if you consider immunodeficiency likely answer is controversial! as some say toxoplasmosis is the most common and some say Cryptococcus is the most common.
Q.126. In lyme disease, which complication is most likely to lead to collapse?
Correct Answer : B
Cardiac complications of Lyme disease generally occur in the early phase and include conduction system disturbances, myopericarditis and congestive heart failure.
Q.127. A 20-year-old student who recently visited Asia came to the OPD with complains of low grade fever, night sweats, anorexia and productive cough. Investigations: CXR cavitatory lesions in upper lobes. What is the most likely causative organism?
Correct Answer : C
Low-grade fever, night sweats, anorexia, dry (initially) or productive cough, and upper lobe cavitary lesions are highly suggestive of TB.
Q.128. Which virus is transmitted by the fecal-oral route?
Correct Answer : B
Coxsackie virus
Q.129. A 7-year-old child presents with lesions on the trunk. Examination: some lesions are weeping and others are crusted with a red base. What is the causative organism?
Correct Answer : B
Chicken pox is centripetal infection where rashes are predominant on trunk and face and less in extremities. There are lesions of different stages. In children herpes zoster is unlikely and chicken pox is common.
Q.130. A middle aged Asian presents with episodes of fever with rigors and chills for last 1 year. Blood film: ring form of plasmodium with schuffners dots in RBCs. What is the drug to eradicate this infection?
Correct Answer : A
Shuffners dot indicates either vivax or ovale infection and the hepatic cycle only can be eradicated by primaquine.
Q.131. A pregnant woman returns from Sudan, now presenting with intermittent fever, rigor and seizures. What is the diagnosis?
Correct Answer : B
Intermittent fever is seen in malaria. In meningitis, fever is not intermittent. Also, Sudan is endemic for malaria.
Q.132. A man was bitten by a drug addict and comes to the hospital with a wound. What investigation should be undertaken?
Correct Answer : C
Hepatitis B
Q.133. A patient of tuberculous abscess with the history of previous abscess drainage presented with fever and tenderness between L2/L3 vertebra. Which is the best investigation for this patient?
Correct Answer : D
To exclude any spinal cord pathology.
Q.134. A patient comes back from India and presents with night sweats and lymphadenopathy. CXR: Cavitation. What investigation should be done next?
Correct Answer : B
The symptoms and arrival from an endemic area for Pulmonary TB suggest the best course of action would be to go for AFB staining via ZN stain.
Q.135. A 16-year-old girl has had an enlarging mass in the right side of her neck for the last 6 weeks. She has had no other symptoms. She has a 2 x 2 cm enlarged LN in the anterior triangle of the neck with several smaller associated LN palpable. Oropharyngeal examination shows tonsillar membranes. What is the most likely diagnosis?
Correct Answer : A
Though in infectious mononucleosis lymph nodes are usually seen in a posterior triangle but can be seen in the whole body including the anterior triangle also.
Q.136. A 22-year-old has had recent chickenpox. He now presents with confusion. He is noted to have low urine output and large petechiae all over his body. CXR: a large patch of consolidation is seen. What is the management of choice?
Correct Answer : D
IV acyclovir
Q.137. Syphilis typically causes which of the following?
Correct Answer : D
Orchitis
Q.138. A 45-year-old man has a smooth, tender swelling extending from the ear to the angle of the jaw of sudden onset. He has a temperature of 38.5 C. What is the most likely diagnosis?
Correct Answer : D
Mumps
Q.139. What is the most appropriate antibiotic to treat uncomplicated chlamydia infection in a 21-year-old female who is not pregnant?
Correct Answer : D
Doxycycline
Q.140. A 28-year-old man has developed a red, raised rash on trunk after playing football. His past medical history shows he had childhood asthma. The rash is becoming increasingly itchy. What is the most appropriate treatment?
Correct Answer : A
Diagnosis is Atopy (allergy).
Q.141. An otherwise healthy 13-year-old boy presents with recurrent episodes of facial and tongue swelling and abdominal pain. His father has had similar episodes. What is the most likely diagnosis?
Correct Answer : A
Diagnosis is hereditary angioedema. Acquired angioedema usually manifests after the age of 40 yrs.
Q.142. A 20-year-old man complains of recent onset of itching which followed a viral infection. There are numerous wheals of all sizes on his skin particularly after he has scratched it. These can last up to an hour. What is the most probable diagnosis?
Correct Answer : B
Urticaria
Q.143. A 19-year-old boy complains of itching on the site of insect bite. What is the most appropriate management?
Correct Answer : C
Oral antihistamine
Q.144. After eating a cookie at a garden party, a child began to cough and went blue. The mother also noticed that there were swollen patches on the skin. What is the diagnosis?
Correct Answer : A
Allergic reaction
Q.145. After eating a cookie at a garden party, a child began to cough and went blue. The mother also noticed that there were swollen patches on the skin. What is the initial management?
Correct Answer : D
Anaphylaxis with partially blocked airway.
Q.146. A 24-year-old patient presented with anaphylactic shock. What would be the dose of adrenaline?
Correct Answer : A
In anaphylaxis adrenaline 0.5ml of 1:1000 is given IM whereas in cardiac arrest 1 ml of 1:1000.
Q.147. A patient presented after eating a seafood dish at a local restaurant. He complains of difficulty in breathing. His speech is slurred and his BP is 85/55mmHg. What would be the most appropriate next step?
Correct Answer : B
Patient is getting anaphylactic shock with the difficulty of breathing and slurred speech (probably from tongue swelling) which is an indication of IM adrenaline.
Q.148. A 37-year-old female working as a healthcare assistant in a nursing home comes to the ED with complaints of severe itching all over her body. On asking she replies that she had applied cream on the body of a resident in the nursing home who had similar itches. What is the mechanism of itching?
Correct Answer : A
Allergic reaction
Q.149. A patient presents with purple papular lesions on his face and upper trunk measuring 1-2 cm across. They aren’t painful or itchy. What is the most likely diagnosis?
Correct Answer : A
It is a spindle-cell tumour derived from capillary endothelial cells or from fibrous tissue, caused by human herpes virus. It presents as purple papules (½ to 1 cm) or plaques on skin and mucosa (any organ). It metastasizes to nodes.
Q.150. A Patient presents with fever, dry cough and breathlessness. He is tachypneic but chest is clear. Oxygen saturation is normal at rest but drops on exercise. What is the most likely diagnosis?
Correct Answer : C
Fever, dry cough, breathlessness, tachypnoea with clear chest is seen in pneumocystis carinii pneumonia. Normal oxygen saturation which drops on exercise is characteristic of pneumocystis carinii pneumonia.
Q.151. A 40-year-old man with marked weight loss over the preceeding 6months has bilateral white, vertically corrugated lesion on the lateral surfaces of the tongue. What is the most likely diagnosis?
Correct Answer : C
The lesion described is leukoplakia which is likely association of HIV disease.
Q.152. A 29-year-old man presents with history of cough, weight loss and night sweats. Exam: pansystolic murmur. What is the most appropriate diagnosis of underlying cause?
Correct Answer : C
Though not mentioned IV drug abusers may have tricuspid regurgitation causing pan systolic murmur and HIV is also very common in this group.
Q.153. A 32-year-old homosexual comes with history of weight loss. Fundoscopy reveals retinal hemorrhages. What is the most appropriate option?
Correct Answer : B
Weight loss in a homosexual is likely to be due to AIDS and CMV retinopathy with retinal hemorrhage is a recognized association.
Q.154. A 30-year-old patient came to the OPD with complaints of breathlessness and dry cough. He has lost 5 kgs in 2 months. He is an IV drug abuser. Investigation: CXR bilateral interstitial shadowing. What is the most likely causative organism?
Correct Answer : D
IV drug abuser, weight loss, breathlessness and dry cough with interstitial shadowing indicate PCP pneumonia in an AIDS patient.
Q.155. A 34-year-old IVDA (intravenous drug abuser) presents with a 4 months history of productive cough. He has lost 10kgs. What is the most appropriate investigation?
Correct Answer : A
In IVDA immunity becomes low and easily gets infected with TB.
Q.156. A HIV +ve 55-year-old man presents with painless lymphadenopathy, fever, night sweats and weight loss. What is the most probable diagnosis?
Correct Answer : B
NHL is more likely diagnosis in AIDS or immunodeficient state.
Q.157. A 38-year-old woman with hemophilia who received several blood transfusions a few years ago presents with irritability and increasing memory deficit. She is unable to speak properly. She is on anti-TB treatment. What is the most likely diagnosis?
Correct Answer : D
Blood transfusion is the clue for HIV transmission. Immunodeficiency is also responsible for TB.
Q.158. A 56-year-old man with known HIV presents with chronic diarrhea. What is the most likely cause?
Correct Answer : D
In AIDS patients common cause of diarrhea is cryptosporidium.
Q.159. A 36-year-old lady who is an intravenous drug user has had weight loss for the past 4 months. He now presents with drowsiness and increasing confusion. What is the most likely cause?
Correct Answer : A
Case of HIV. Cryptococcus can cause meningitis in AIDS patients
Choose a Question
×Choose a Question
×
Trial Access Limit Reached
You’ve reached the limit of free content. Subscribe to continue learning without restrictions.